Stay ahead of learning milestones! Enroll in a class over the summer!

G
Topic
First Poster
Last Poster
variable point on the line BC
orl   26
N 4 minutes ago by Ilikeminecraft
Source: IMO Shortlist 2004 geometry problem G7
For a given triangle $ ABC$, let $ X$ be a variable point on the line $ BC$ such that $ C$ lies between $ B$ and $ X$ and the incircles of the triangles $ ABX$ and $ ACX$ intersect at two distinct points $ P$ and $ Q.$ Prove that the line $ PQ$ passes through a point independent of $ X$.
26 replies
orl
Jun 14, 2005
Ilikeminecraft
4 minutes ago
Geometry from EGMO 2018
BarishNamazov   35
N an hour ago by math-olympiad-clown
Source: EGMO 2018 P1
Let $ABC$ be a triangle with $CA=CB$ and $\angle{ACB}=120^\circ$, and let $M$ be the midpoint of $AB$. Let $P$ be a variable point of the circumcircle of $ABC$, and let $Q$ be the point on the segment $CP$ such that $QP = 2QC$. It is given that the line through $P$ and perpendicular to $AB$ intersects the line $MQ$ at a unique point $N$.
Prove that there exists a fixed circle such that $N$ lies on this circle for all possible positions of $P$.
35 replies
BarishNamazov
Apr 11, 2018
math-olympiad-clown
an hour ago
CMI Entrance 19#4
bubu_2001   13
N 2 hours ago by Mathworld314
Let $ABCD$ be a parallelogram $.$ Let $O$ be a point in its interior such that $\angle AOB + \angle DOC = 180^{\circ} . $
Show that $,\angle ODC = \angle OBC . $
13 replies
bubu_2001
Oct 31, 2019
Mathworld314
2 hours ago
Reducibility of 2x^2 cyclotomic
vincentwant   3
N 3 hours ago by YaoAOPS
Let $S$ denote the set of all positive integers less than $1020$ that are relatively prime to $1020$. Let $\omega=\cos\frac{\pi}{510}+i\sin\frac{\pi}{510}$. Is the polynomial $$\prod_{n\in S}(2x^2-\omega^n)$$reducible over the rational numbers, given that it has integer coefficients?
3 replies
vincentwant
Apr 30, 2025
YaoAOPS
3 hours ago
inequalities
Tamako22   2
N 3 hours ago by Tamako22
let $a,b,c> 1,\dfrac{1}{1+a}+\dfrac{1}{1+b}+\dfrac{1}{1+c}=2.$
prove that$$\sqrt{a}+\sqrt{b}+\sqrt{c}\ge \dfrac{2}{\sqrt{a}}+\dfrac{2}{\sqrt{b}}+\dfrac{2}{\sqrt{c}}$$
2 replies
Tamako22
Yesterday at 12:18 PM
Tamako22
3 hours ago
Functional Equation
Keith50   2
N 3 hours ago by jasperE3
Source: Own
Find all functions $f:\mathbb{R} \rightarrow \mathbb{R}$ such that \[f(x+f(x)+2f(y))+f(2f(x)-y)=4x+f(y)\]holds for all reals $x$ and $y$.
2 replies
Keith50
Jun 24, 2021
jasperE3
3 hours ago
3-var inequality
sqing   2
N 3 hours ago by sqing
Source: Own
Let $ a,b,c\geq 0 ,a+b+c =4. $ Prove that
$$a +ab^2 +ab^2c \leq\frac{33}{4}+2\sqrt 2$$$$a +ab^2 +abc \leq \frac{2(100+13\sqrt {13})}{27}$$$$a +a^2b + a b^2c^3\leq \frac{2(82+19\sqrt {19})}{27}$$
2 replies
sqing
Yesterday at 3:56 AM
sqing
3 hours ago
Popular children at camp with algebra and geometry
Assassino9931   2
N 3 hours ago by cj13609517288
Source: RMM Shortlist 2024 C3
Fix an odd integer $n\geq 3$. At a maths camp, there are $n^2$ children, each of whom selects
either algebra or geometry as their favourite topic. At lunch, they sit at $n$ tables, with $n$ children
on each table, and start talking about mathematics. A child is said to be popular if their favourite
topic has a majority at their table. For dinner, the students again sit at $n$ tables, with $n$ children
on each table, such that no two children share a table at both lunch and dinner. Determine the
minimal number of young mathematicians who are popular at both mealtimes. (The minimum is across all sets of topic preferences and seating arrangements.)
2 replies
Assassino9931
Friday at 11:07 PM
cj13609517288
3 hours ago
Interesting inequalities
sqing   1
N 4 hours ago by sqing
Source: Own
Let $ a,b> 0 $ and $  a^2+ab+b^2=a+b   $. Prove that
$$   \frac{a }{2b^2+1}+ \frac{b }{2a^2+1}+ \frac{1}{2ab+1} \geq \frac{21}{17}$$Let $ a,b> 0 $ and $ a^2+ab+b^2=a+b+1   $. Prove that
$$   \frac{a }{2b^2+1}+ \frac{b }{2a^2+1}+ \frac{1}{2ab+1} \geq1$$
1 reply
sqing
4 hours ago
sqing
4 hours ago
Interesting inequalities
sqing   0
4 hours ago
Source: Own
Let $ a,b> 0 $ and $  a^2+ab+b^2=k(a+b)   $. Prove that
$$   \frac{a }{2b^2+1}+ \frac{b }{2a^2+1}+ \frac{1}{2ab+1} \geq \frac{12k+9}{8k^2+9}$$Where $ k\in N^+.$
Let $ a,b> 0 $ and $  a^2+ab+b^2=3(a+b)   $. Prove that
$$   \frac{a }{2b^2+1}+ \frac{b }{2a^2+1}+ \frac{1}{2ab+1} \geq \frac{5}{9}$$
0 replies
sqing
4 hours ago
0 replies
Queue geo
vincentwant   6
N 4 hours ago by ihategeo_1969
Let $ABC$ be an acute scalene triangle with circumcenter $O$. Let $Y, Z$ be the feet of the altitudes from $B, C$ to $AC, AB$ respectively. Let $D$ be the midpoint of $BC$. Let $\omega_1$ be the circle with diameter $AD$. Let $Q\neq A$ be the intersection of $(ABC)$ and $\omega$. Let $H$ be the orthocenter of $ABC$. Let $K$ be the intersection of $AQ$ and $BC$. Let $l_1,l_2$ be the lines through $Q$ tangent to $\omega,(AYZ)$ respectively. Let $I$ be the intersection of $l_1$ and $KH$. Let $P$ be the intersection of $l_2$ and $YZ$. Let $l$ be the line through $I$ parallel to $HD$ and let $O'$ be the reflection of $O$ across $l$. Prove that $O'P$ is tangent to $(KPQ)$.
6 replies
vincentwant
Apr 30, 2025
ihategeo_1969
4 hours ago
Problem 6
SlovEcience   3
N 4 hours ago by Tung-CHL
Given two points A and B on the unit circle. The tangents to the circle at A and B intersect at point P. Then:
\[ p = \frac{2ab}{a + b} \], \[ p, a, b \in \mathbb{C} \]
3 replies
SlovEcience
Yesterday at 9:44 AM
Tung-CHL
4 hours ago
parallel lines with incenter
Scorpion.k48   6
N Mar 1, 2018 by liekkas
Let $\triangle ABC$ with incenter $I$. $AI, BI, CI$ cuts $BC, CA, AB$ at $D, E, F$, res. Let $X$ is orthocenter of $\triangle DEF$. Prove that $IX$ is parallel to Euler line of $\triangle ABC$
6 replies
Scorpion.k48
Aug 5, 2015
liekkas
Mar 1, 2018
parallel lines with incenter
G H J
G H BBookmark kLocked kLocked NReply
The post below has been deleted. Click to close.
This post has been deleted. Click here to see post.
Scorpion.k48
195 posts
#1 • 2 Y
Y by Adventure10, Mango247
Let $\triangle ABC$ with incenter $I$. $AI, BI, CI$ cuts $BC, CA, AB$ at $D, E, F$, res. Let $X$ is orthocenter of $\triangle DEF$. Prove that $IX$ is parallel to Euler line of $\triangle ABC$
Z K Y
The post below has been deleted. Click to close.
This post has been deleted. Click here to see post.
A-B-C
254 posts
#2 • 1 Y
Y by Adventure10
It has been posted here http://www.artofproblemsolving.com/community/c6h171583
You can see the proof of Jean-Louis Ayme in his website.
Z K Y
The post below has been deleted. Click to close.
This post has been deleted. Click here to see post.
TelvCohl
2312 posts
#3 • 9 Y
Y by mineiraojose, A-B-C, enhanced, bobaboby1, noway, AlastorMoody, tarzanjunior, enzoP14, Adventure10
My solution :

Lemma 1 :

Let $ I $ be the incenter of $ \triangle ABC $ and $ \triangle XYZ $ be the incentral triangle of $ \triangle ABC $ .
Let $ D, E, F $ be the reflection of $ I $ in $ BC, CA, AB $, resp and $ P \equiv AD \cap BE \cap CF $ .

Then $ IP $ passes through the orthocenter $ H $ of $ \triangle XYZ $

Proof :

Let $ Q \equiv YH \cap ZX, R \equiv XH \cap YZ, S \equiv AX \cap YZ $ .
Let $ T \equiv YZ \cap BC, U \equiv AD \cap BC, V \equiv BE \cap CA $ .

From $ U(A,I;S,X)=-1 $ and $ \angle IUX=\angle DUX \Longrightarrow US \perp UX \equiv BC $ ,
so combine $ AT \perp AI \Longrightarrow A, S, T, U $ are concyclic $ \Longrightarrow \angle IUS=\angle AUS=\angle ATS $ ,
hence notice $ A, R, T, X $ are concyclic $ \Longrightarrow \angle IUS=\angle ATS=\angle AXR=\angle RUS \Longrightarrow I \in UR $ ,
so from Desargue theorem (for $ \triangle ABU $ and $ \triangle XYR $) $ \Longrightarrow XR \cap AP $ lie on the antiorthic axis of $ \triangle ABC $ .

Similarly, we can prove $ I \in VQ $ and $ YQ \cap BP $ lie on the the antiorthic axis of $ \triangle ABC $ ,
so from Desargue theorem (for $ \triangle ABP $ and $ \triangle XYH $) $ \Longrightarrow AX \cap BY \in PH $ . i.e. $ P, I, H $ are collinear
____________________________________________________________
Lemma 2 :

Let $ I $ be the incenter of $ \triangle ABC $ .
Let $ D, E, F $ be the reflection of $ I $ in $ BC, CA, AB $, respectively .

Then $ IP $ is parallel to the Euler line $ \mathcal{E} $ of $ \triangle ABC $ where $ P \equiv AD \cap BE \cap CF $

Proof :

Let $ Y \equiv FD \cap CA, Z \equiv DE \cap AB $ .
Let $ \triangle A_1B_1C_1, \triangle A_2B_2C_2 $ be the intouch triangle, orthic triangle of $ \triangle ABC $, respectively .
Let $ T $ be the midpoint of $ AI $ and $ \overline{A_3B_3C_3} $ be the orthic axis of $ \triangle ABC $ ($ A_3\in BC, B_3\in CA,C_3\in AB $) .

Since $ \odot (Y, YI), \odot (Z, ZI) $ is the image of $ BE, CF $, resp under the inversion $ \mathbf{I}(\odot (DEF)) $ ,
so $ IP $ is the radical axis of $ \odot (Y, YI), \odot (Z, ZI) \Longrightarrow IP \perp YZ $ (also follows from Sondat theorem) . (1)

From $ TC_1 \parallel AF, A_1C_1 \parallel DF \Longrightarrow \angle AFD=\angle TC_1A_1 $ ,
so notice $ T $ is the center of $ \odot (AI) \Longrightarrow \angle AFD=90^{\circ}-\tfrac{1}{2} \angle A+\tfrac{1}{2} \angle B $ ,

hence by sine rule we get $ AY=AF \cdot \frac{\sin \angle AFD}{\sin \angle AYD}=AI \cdot \frac{\cos \tfrac{1}{2} (\angle A-\angle B)}{\sin \tfrac{1}{2} (\angle A-\angle C)} $ .

Similarly, $ AZ=AI \cdot \frac{\cos \tfrac{1}{2} (\angle A-\angle C)}{\sin \tfrac{1}{2} (\angle A-\angle B)} \Longrightarrow \frac{AY}{AZ}=\frac{ \sin \tfrac{1}{2} (\angle A-\angle B) \cdot \cos \tfrac{1}{2} (\angle A-\angle B)}{\sin \tfrac{1}{2} (\angle A-\angle C) \cdot \cos \tfrac{1}{2} (\angle A-\angle C)}=\frac{\sin (\angle A-\angle B)}{\sin (\angle A-\angle C)} $ . (2)

Since $ AB_3=AC_2 \cdot \frac{\sin \angle AC_2B_3}{\sin \angle AB_3C_2}= \frac{AC_2 \cdot \sin \angle C}{\sin (\angle A-\angle C) } , AC_3=AB_2 \cdot \frac{\sin \angle AB_2C_3}{\sin \angle AC_3B_2}=\frac{AB_2 \cdot \sin \angle B}{\sin (\angle A-\angle B) } $ ,

so we get $ \frac{AB_3}{AC_3}=\frac{AC_2 \cdot \sin \angle C}{AB_2 \cdot \sin \angle B} \cdot \frac{\sin (\angle A-\angle B)}{\sin (\angle A-\angle C)}=\frac{AC_2 \cdot AB}{AB_2 \cdot AC} \cdot \frac{\sin (\angle A-\angle B)}{\sin (\angle A-\angle C)}=\frac{\sin (\angle A-\angle B)}{\sin (\angle A-\angle C)} $ ,

hence combine (2) $ \Longrightarrow B_3C_3 \parallel YZ \Longrightarrow YZ \perp \mathcal{E} $ ($\because B_3C_3 $ is perpendicular to $ \mathcal{E} $ (well-known)) $ \Longrightarrow IP \parallel \mathcal{E} $ (from (1)) .
____________________________________________________________
Back to the main problem :

Let $ I_a, I_b, I_c $ be the reflection of $ I $ in $ BC, CA, AB $, resp and $ P \equiv AI_a \cap BI_b \cap CI_c $ .

From lemma 1 $ \Longrightarrow P \in IX $, so from lemma 2 $ \Longrightarrow IX \equiv IP $ is parallel to the Euler line of $ \triangle ABC $ .

Q.E.D
____________________________________________________________
P.S. The fact $ AI_a, BI_b, CI_c $ are concurrent simplified follows from Jacobi theorem :)
Attachments:
This post has been edited 1 time. Last edited by TelvCohl, Aug 5, 2015, 11:45 PM
Z K Y
The post below has been deleted. Click to close.
This post has been deleted. Click here to see post.
A-B-C
254 posts
#4 • 3 Y
Y by bobaboby1, Adventure10, Mango247
My solution is based on some properties that appearred on ETC.
$X=X_{500}$, $I=X_1$
From http://faculty.evansville.edu/ck6/encyclopedia/glossary.html and http://faculty.evansville.edu/ck6/encyclopedia/ETC.html(see $X_{35},X_{79},X_{500}$, we have $X_{500}$ is crosssum of $X_{1},X_{79}$ so $X_{500},X_1,X_{79}$ are collinear.
So we have to show that $X_1X_{79}$ is parallel to Euler line of $\triangle ABC$.
Also from here http://faculty.evansville.edu/ck6/encyclopedia/ETC.html, if $A',B',C'$ are reflections of $I$ in $BC,CA,AB$ then $AA',BB',CC'$ are concurrent at $X_{79}$.
From properties of Neuberg cubic,http://bernard.gibert.pagesperso-orange.fr/Exemples/k001.html, $P$ lies on Neuberg cubic, $P_a,P_b,P_c$ are reflections of $P$ in $BC,CA,AB$ then $AP_a,BP_b,CP_c$ are concurrent at a point on $PP*$ where $P*$ is isogonal conjugate of $P$ WRT $\triangle ABC$, furthermore, $PP*$ is parallel to Euler line of $\triangle ABC$ so $X_{1}X_{79}\parallel X_{3}X_{4}$.
Z K Y
The post below has been deleted. Click to close.
This post has been deleted. Click here to see post.
A-B-C
254 posts
#5 • 4 Y
Y by mineiraojose, bobaboby1, Adventure10, Mango247
This proof is wrong.
______
A synthetic proof for lemma 2 in Telv's post:
$H_a$is orthocenter of $\triangle IBC$, $AI$ intersects $\odot (ABC)$ at $O_a\ne A$ then $O_a$ is circumcenter of $\triangle IBC$
$O_b,O_c$ is determined similarly.
Since $BH_a,CH_a\perp IC,IB$, and $IO_{c}=BO_c$, $IO_{b}=CO_b$ then $H_a$ is circumcenter of $\triangle IO_bO_c$
Similarly $\Rightarrow$ $O_aH_a$, $O_bH_b$, $O_cH_c$ are concurrent at Kosnita point $S$ of $\triangle O_aO_bO_c$(also Schiffler point and Schiffler point lie on Euler line)
$A,B,C$ are circumcenters of $\triangle IEF,\triangle IFD,\triangle IDE$ so $AD,BE,CF$ are concurrent at Kosnita point $P$ of $\triangle DEF$
$\triangle DEF$ and $\triangle O_aO_bO_c$ are homothetic, $I$, $O$ are their circumcenters
$\Longrightarrow$ $IP\parallel OS$, or $IP$ is parallel to Euler line of $\triangle ABC$
Attachments:
This post has been edited 3 times. Last edited by A-B-C, Dec 6, 2021, 4:45 AM
Reason: wrong
Z K Y
The post below has been deleted. Click to close.
This post has been deleted. Click here to see post.
TelvCohl
2312 posts
#6 • 6 Y
Y by mineiraojose, bobaboby1, enzoP14, enhanced, Adventure10, Mango247
TelvCohl wrote:
Lemma 2 :

Let $ I $ be the incenter of $ \triangle ABC $ .
Let $ D, E, F $ be the reflection of $ I $ in $ BC, CA, AB $, respectively .

Then $ IP $ is parallel to the Euler line $ \mathcal{E} $ of $ \triangle ABC $ where $ P \equiv AD \cap BE \cap CF $
Another proof to the Lemma 2 at post #3 :

Lemma :

Let $ \triangle DEF $ be the orthic triangle of $ \triangle ABC $ and $ Y $ $ \equiv $ $ CA $ $ \cap $ $ FD, $ $ Z $ $ \equiv $ $ AB $ $ \cap $ $ DE $. Let $ T $ be the intersection of the bisector of $ \angle DYC $ and $ \angle DZB $. Let $ I_a $ be the reflection of the Incenter $ I $ of $ \triangle ABC $ in $ BC $. Then $ A, $ $ T, $ $ I_a $ are collinear.

Proof :

Let $ K $ $ \equiv $ $ AI_a $ $ \cap $ $ BC, $ $ U $ $ \equiv $ $ BI $ $ \cap $ $ CA, $ $ V $ $ \equiv $ $ CI $ $ \cap $ $ AB $. Let $ P, $ $ Q $ be the projection of $ U $ on $ AB, $ $ BC $, respectively and $ R, $ $ S $ be the projection of $ V $ on $ BC, $ $ CA $, respectively. From $ AP $ $ / $ $ FP $ $ = $ $ AU $ $ / $ $ CU $ $ = $ $ AB $ $ / $ $ CB $ $ = $ $ AD $ $ / $ $ CF $ $ = $ $ AY $ $ / $ $ FY $ we get $ YP $ is the bisector of $ \angle DYC $, so $ Y, $ $ T, $ $ P $ are collinear. Similarly, we can prove $ Q $ $ \in $ $ YT, $ $ R $ $ \in $ $ ZT $ and $ S $ $ \in $ $ ZT $.

Let $ T_1 $ $ \equiv $ $ PQ $ $ \cap $ $ AI_a, $ $ T_2 $ $ \equiv $ $ RS $ $ \cap $ $ AI_a $. From Menelaus theorem (for $ \triangle ABK $ and $ \overline{QT_1P} $) and notice $ BP $ $ = $ $ BQ $ we get $ AT_1 $ $ / $ $ KT_1 $ $ = $ $ AP $ $ / $ $ KQ $ ... (1). Similarly, we can prove $ AT_2 $ $ / $ $ KT_2 $ $ = $ $ AS $ $ / $ $ KR $ ... (2). On the other hand, if $ G $ $ \equiv $ $ AI $ $ \cap $ $ UV $ then from $ (KA, KI; KG, BC)=-1 $ and $ BC $ bisect $ \angle AKI $ we get $ GK $ $ \perp $ $ BC $, so $ KQ $ $ / $ $ KR $ $ = $ $ GU $ $ / $ $ GV $ $ = $ $ AU $ $ / $ $ AV $ $ = $ $ AP $ $ / $ $ AS $, hence combine with (1) and (2) we conclude that $ T_1 $ $ \equiv $ $ T_2 $ $ \equiv $ $ T $. i.e. $ A, $ $ T, $ $ I_a $ are collinear
____________________________________________________________
Back to the main proof :

Let $ \triangle XYZ $ be the orthic triangle of $ \triangle ABC $. Let $ X_1 $ be the intersection of the bisector of $ \angle (CA, ZX) $ and the bisector of $ \angle (AB, XY) $ (define $ Y_1, $ $ Z_1 $, similarly). From the Lemma $\Longrightarrow $ $ X_1 $ $ \in $ $ AD, $ $ Y_1 $ $ \in $ $ BE $ and $ Z_1 $ $ \in $ $ CF $, so the perspectrix of any two triangles of $ \{ \triangle ABC, \triangle DEF, \triangle X_1Y_1Z_1 \} $ are concurrent, hence the perspectrix $ \tau $ of $ \triangle ABC, $ $ \triangle DEF $ is parallel to the Orthic axis of $ \triangle ABC $ (notice $ \triangle DEF $ and $ \triangle X_1Y_1Z_1 $ are homothetic).

From Sondat's theorem we get $ \tau $ is perependicular to the line connecting the orthology center $ I $ of $ \{ \triangle ABC, \triangle DEF \} $ and the perspector $ P $ of $ \triangle ABC, $ $ \triangle DEF $, so we conclude that $ IP $ is parallel to the Euler line of $ \triangle ABC $.
Z K Y
The post below has been deleted. Click to close.
This post has been deleted. Click here to see post.
liekkas
370 posts
#7 • 3 Y
Y by bobaboby1, Adventure10, Mango247
A-B-C wrote:
$BI=BO_c,CI=CO_b$

Thanks for your reply, but you simply went wrong here. Please check it carefully:)
This post has been edited 1 time. Last edited by liekkas, Mar 1, 2018, 4:45 PM
Z K Y
N Quick Reply
G
H
=
a